Author Topic: EM Drive Developments - related to space flight applications - Thread 3  (Read 3130954 times)

Offline Rodal

  • Senior Member
  • *****
  • Posts: 5911
  • USA
  • Liked: 6124
  • Likes Given: 5564
...

Also Dr. Rodal:
Doesn't a TM mode require the RF feed from the end plates? Or does a magnetron antenna work differently?

Great question.

First let me start by stating that I don't think :) that one can excite (in general, for higher modes where there are a lot of eigenfrequencies next to each other) this or that mode shape by simple positioning of the RF source, a lot depends on the spectrum of frequencies being excited and it depends on the eigenfrequencies of the modes that are nearby.  A great source of good technical information for this is CERN that has very sophisticated cavities used for resonance.  In general it is unavoidable that what one is going to get is a superposition of mode shape excitations at any frequency.  So two points here:

1) one is never going to excite a single mode [particularly when using a magnetron that puts out quite a spectrum of frequencies], the most that one can aim for is maximizing the response for one mode and minimizing the response of other modes

2) to calculate the amount of participation of different modes in the resonance, one would have to conduct a spectrum analysis (this can be done with ANSYS, COMSOL, ADINA, and other finite element codes).  All the COMSOL analysis I have seen reported from NASA (thanks to Star-Drive !!!) have been eigenvalue analyses.  I have never seen (from NASA and much less from anyone else) a spectrum analysis response done for the EM Drive.

My understanding is that NASA uses their S21 plot and S11 plots to know when they are in resonance, and one assumes that the closest eigenvalues to the excitation frequency are the ones being excited.

Also, for their tests at frequencies lower than 2 GHz with a dielectric, NASA confirmed that they were exciting mainly TM212 mode with their thermal camera (that confirms that TM212 had the greatest participation in heating of the big end).

Having said that, I assumed (dangerously ) :) that TM212 is excited to a considerable extent because it is the closest eigenvalue at ~2.45GHz.  There is another TM mode close to 2.45GHz: TM311, occurring at a little lower frequency.  The TE modes eigenfrequencies are further away. I ( dangerously :) ) think that although the RF feed may not be optimal to excite TM modes, as you said, the possibility for exciting the TE modes (that are much significantly further away in frequency) at 2.45 GHz is even more remote (*)

To know exactly what modes are being excited and what is their participation in the response one needs, not only to take into account the geometrical location of the RF Feed, and the spectrum excited by the magnetron but also to conduct a spectrum analysis of the response taking into account all the eigenfrequencies nearby.

__________

(*) This will of course change with Iulian being able to change the length of his EM Drive with the gear mechanism he is working on that will allow variable length.  However I have not calculated what he is going to be able to achieve with the length he has available to change.
« Last Edit: 06/03/2015 07:03 pm by Rodal »

Offline WarpTech

  • Full Member
  • ****
  • Posts: 1407
  • Do it!
  • Statesville, NC
  • Liked: 1453
  • Likes Given: 1925
...
Regarding space-time. Believe me when I tell you, what we are doing here with attenuation and superposition of EM waves "is" modifying space-time at this narrow bandwidth of the EM spectrum. It is NOT in any way going to generate a gravitational field where clocks, rulers, or lasers will be affected. What is going on at the microwave frequencies close to the cut-off is conceptually, and physically the same as-if space-time were curved at these frequencies. If you try to curve space-time at "all" the frequencies of the ZPF spectrum, such that it affects protons, etc.., the amount of energy required is enormous, as GR will tell you. Having a frequency dependent metric is something I predicted at least 15 years ago, when I first started modeling the PV as a quantum field theory. I had no idea how to do it back then. Now we do! :)

Todd

I like where you are going with this.

But what do you make of Eaglework's results, when they measured "space-time contractions" using a 632.8 nm laser? Isn't this far from bandwidth of the RF signal used?   
http://forum.nasaspaceflight.com/index.php?topic=36313.msg1362403#msg1362403
http://forum.nasaspaceflight.com/index.php?topic=36313.msg1355764#msg1355764

Also Dr. Rodal:
Doesn't a TM mode require the RF feed from the end plates? Or does a magnetron antenna work differently?

IMO, what they measured is most likely a change in the refractive index of the air inside the pillbox due to excitation of water vapor by the microwaves. Just like heat rising off a road distorts the reflected light, this would affect a laser beam. Dr. White really has no clue at this stage in the experiments.

Offline Space Ghost 1962

  • Senior Member
  • *****
  • Posts: 2780
  • Whatcha gonna do when the Ghost zaps you?
  • Liked: 2925
  • Likes Given: 2247
...

Also Dr. Rodal:
Doesn't a TM mode require the RF feed from the end plates? Or does a magnetron antenna work differently?

Great question.

First let me start by stating that I don't agree  :) with posters in this thread that claim that they are going to be exciting this or that mode shape by simple positioning of the RF source.  A great source of good technical information for this is CERN that has very sophisticated cavities used for resonance.  In general it is unavoidable that what one is going to get is a superposition of mode shape excitations at any frequency.  So two points here:

1) one is never going to excite a single mode, the most that one can aim for is maximizing the response for one mode and minimizing the response of other modes

2) to calculate the amount of participation of different modes in the resonance, one would have to conduct a spectrum analysis (this can be done with ANSYS, COMSOL, ADINA, and other finite element codes).  All the COMSOL analysis I have seen reported from NASA (thanks to Star-Drive !!!) have been eigenvalue analyses.  I have never seen (from NASA and much less from anyone else) an spectrum analysis response done for the EM Drive.

My understanding is that NASA uses their S21 plot and S11 plots to know when they are in resonance, and one assumes that the closest eigenvalues to the excitation frequency are the ones being excited.

Also, for their tests at frequencies lower than 2 GHz with a dielectric, NASA confirmed that they were exciting mainly TM212 mode with their thermal camera (that confirms that TM212 had the greatest participation in heating of the big end).

Having said that, I assumed (living dangereously) :) that TM212 is excited to a considerable extent because it is the closest eigenvalue at ~2.45GHz.  There is another TM mode close to 2.45GHz: TM311, occurring at a little lower frequency.  The TE modes eigenfrequencies are much further away. I (living dangereously :) ) think that although the RF feed may not be optimal to excite TM modes, as you said, the possibility for exciting the TE modes (that are much further away in frequency) at 2.45 GHz is even more remote (*)

To know exactly what modes are being excited and what is their participation in the response one needs, not only to take into account the geometrical location of the RF Feed, but also to conduct a spectrum analysis of the response taking into account all the eigenfrequencies nearby.

__________

(*) This will of course change with Iulian being able to change the length of his EM Drive with the gear mechanism he is working on that will allow variable length.  However I have not calculated what he is going to be able to achieve with the length he has available to change.

Very, very important.

BTW, there should be a correlation between this and any theory of propulsion, because it's the "lever arm" so to speak for propulsion magnitude. If it doesn't correlate, it's not an EM effect.

So this allows you to get to the bottom of all of this. Once so, you can follow a chain of evidence to falsify theories convincingly, leaving a vastly reduced set to go thru. Goes back to an environment any microwave engineer (or propulsion engineer ;) ) can sort out.

Offline MyronQG

  • Member
  • Posts: 14
  • Liked: 9
  • Likes Given: 5
...
If it can help, here you find and old and "classical" demonstration of the impossibility of accelerating a closed system by converting the system internal energy into kinetic energy. Let's assimilate the spacecraft (including thruster, PPU, a.s.o.) to a particle having a rest mass m0 and 4-velocity V=(c/alpha, v/alpha) in an arbitrary Lorentzian frame. The 4-momentum of the particle, by definition is P=(mc, mv) with m=mo/alpha; if the particle is isolated (no interaction nor mass/energy exchange with the surrounding medium), then the relativistic conservation of momentum implies dP=(dm c, dm v)=0, i.e., dm=0 and d(m v)=0, so dv=0. Total energy is conserved and 3-momentum is also conserved, the former implying that according to the latter the system cannot accelerate.

If you have a "Lorentzian frame", this is correct. However, in the frustum this is not true because you have a variable refractive index. It is no longer "Lorentzian" because there is an attenuation (acceleration) over a narrow bandwidth of the EM spectrum.

Todd

Just take a marker and draw a spot on the spacecraft hull. The theory works equally well for the motion of that spot, irrespective of what's happening inside, provided nothing is coming out of the enclosing control volume. I hate to say that this is basic theory of propulsion (Koëlle, Handbook of Astronautical Engineering, 1962).

A mass in a gravitational field "falls" yet nothing is coming out of the enclosing control volume either. It falls precisely because the stress energy Tensor in that volume is skewed to one side. This is what is happening inside the frustum, but only over a narrow bandwidth of the EM spectrum.

If I get it right, you are assuming that a gravitational field is induced by the electromagnetic fields within the frustrum, which I concede since EM fields are sources of the Einstein equations. However, back of the envelope calculations (looking for the gravity due to the mass of a huge EM field within a 1 m**3 volume) show that even in the best scenario the expected gravitational thrusts effects are orders of magnitude below what it has been observed (or claimed).  I would like to have a look at a peer-reviewed reference where the amplification mechanism for narrow bandwith of the EM spectrum is duly explained.

Offline deltaMass

  • Full Member
  • ****
  • Posts: 955
  • A Brit in California
  • Liked: 671
  • Likes Given: 275
Well goup, I'm taking the plunge...right now. At the risk of being ridiculed by my wife, and maybe a couple on these threads ;), I have attached a Preliminary rev of my proposed emdrive experiment. I welcome any constructive comments, corrections and suggestions. I am putting it out in rough/prelim form to elicit feedback before I get underway in earnest this summer.

In addition, it might motivate others out there to get busy themselves!

p.s. Special thanks to Zellerium for "inspiration" on the experiment outline...

Note: My approach is not to replicate. I've decided on a lower power model for reasons stated in the paper. Nothing written yet has precluded the use of these lower power levels. Perhaps my humble efforts might help with that aspect.

Dave
Good luck with that! As regards the effect of heated air, you'll find (either by calculation or experiment) that if the device is able to let any air escape or enter, the change in weight due to lost air far exceeds the change in weight to due to buoyancy in air caused by ballooning.

You can do a lot better than 100 mg rez on your weighing device for quite moderate bucks. I bought my used Mettler H20 for $80 online and it gets 10 ug.

I'd recommend a fully mechanical weighing device, so as to avoid e/m effects on its electronics.

Offline rfmwguy

  • EmDrive Builder (retired)
  • Senior Member
  • *****
  • Posts: 2205
  • Liked: 2713
  • Likes Given: 1134
Well goup, I'm taking the plunge...right now. At the risk of being ridiculed by my wife, and maybe a couple on these threads ;), I have attached a Preliminary rev of my proposed emdrive experiment. I welcome any constructive comments, corrections and suggestions. I am putting it out in rough/prelim form to elicit feedback before I get underway in earnest this summer.

In addition, it might motivate others out there to get busy themselves!

p.s. Special thanks to Zellerium for "inspiration" on the experiment outline...

Note: My approach is not to replicate. I've decided on a lower power model for reasons stated in the paper. Nothing written yet has precluded the use of these lower power levels. Perhaps my humble efforts might help with that aspect.

Dave
Good luck with that! As regards the effect of heated air, you'll find (either by calculation or experiment) that if the device is able to let any air escape or enter, the change in weight due to lost air far exceeds the change in weight to due to buoyancy in air caused by ballooning.

You can do a lot better than 100 mg rez on your weighing device for quite moderate bucks. I bought my used Mettler H20 for $80 online and it gets 10 ug.

I'd recommend a fully mechanical weighing device, so as to avoid e/m effects on its electronics.

Thks, One of the reasons I chose to use copper screen mesh for frustum side walls is to negate the ballooning.

Appreciate the scale input, really need a 2000g by 0.01g if I can find such a thing. Mechanical is probably the best, but at low RF levels, thought I'd try digital...cheers.

Offline Rodal

  • Senior Member
  • *****
  • Posts: 5911
  • USA
  • Liked: 6124
  • Likes Given: 5564
the near field propagates faster than light?  I'm not sure I can bring myself to swallow that just yet.


Superluminal Group Velocity of Electromagnetic Near-field
http://arxiv.org/abs/physics/0311061

 :)

There are dozens of papers reporting experiments measuring superluminal propigation of evanescent EM waves. Use Google to find your favorites.

It happens that the solution of the wave equations (Maxwell) are very similar, if not identical to the equations discribing the tunneling phenomona. I wonder if tunneling and superluminal propagation may be in fact the same phenomona? Does anyone know of any experimental measurements of momentum resulting from electron tunneling?
I agree.

SUBJECTIVE OPINION: I much prefer the wording " tunneling phenomena" to "superluminal propagation" as it does not necessarily involve the need to invoke the existence of tachyons. 

QUESTION 1: The group velocity is superluminal.  Is the phase velocity subluminal ?

The number of photons in a photon gas is not a thermodynamic constant but it is proportional to the cube of the temperature of the photon gas. In the Photonic Laser Thruster, collimated photons are reused by mirrors, multiplying the force by the number of bounces. QUESTION 2:   Under Q resonance can the number of photons able to achieve "tunneling phenomena", be greater than the number of photons emitted by a traditional photon rocket?

Trying to find answer to your question, I find that we are not alone in preferring "tunneling phonomena" terminology.
I have posted this reference several times already, but http://wwwsis.lnf.infn.it/pub/INFN-FM-00-04.pdf
Quote
With regard to the same experiments, Guenter Nimitz claimed very recently [10] that those results with evanescent waves, or "tunnelling photons", do really imply Superluminal signal and impulse transmission.
pdf, can't copy/paste so quote is re-typed. Any errors in the quotation are mine.

Page 3, section 2

You will note that on page 15 of this paper, the evanescent wave equation is the same as the tunneling equation.
As for phase velocity and photon count, afraid I've struck out, so far.

(I think that aero was one of the first ones to notice this, a long time ago on Thread 1), no researcher (with the possible exception of Shawyer (*))  has measured a Thrust/PowerInput superior to (the quality factor of resonance) "Q" times the  Thrust/PowerInput of a perfectly collimated photon rocket.

NASA measured a Thrust/PowerInput significantly below Q times the  Thrust/PowerInput of a perfectly collimated photon rocket.

Thus the possibility of quantum tunneling photons being responsible for EM Drive thrust is at play.

It involves no esoteric physics (no 5D branes, no gravitation, no dark energy or dark mass).

Also it neatly deals with conservation of momentum, as the thrust would be just due to the photons.

The thing that remains to be explained is whether quantum tunneling can take place and:

The number of photons in a photon gas is not a thermodynamic constant but it is proportional to the cube of the temperature of the photon gas. In the Photonic Laser Thruster, collimated photons are reused by mirrors, multiplying the force by the number of bounces. QUESTION :   Under Q resonance can the number of photons able to achieve "tunneling phenomena", be greater than the number of photons emitted by a traditional photon rocket?



(*)

Shawyer Demonstration = 53% to 162% of the Q times the Thrust/PowerInput of a perfectly collimated photon rocket (can't be precise because the power reported varied from 421-1200 watts)
ShawyerExperimental    93 % of the Q times the Thrust/PowerInput of a perfectly collimated photon rocket
(NASA Brady a)             22% of the Q times the Thrust/PowerInput of a perfectly collimated photon rocket
(NASA Brady b)                5% of the Q times the Thrust/PowerInput of a perfectly collimated photon rocket
(NASA Brady a)              29% of the Q times the Thrust/PowerInput of a perfectly collimated photon rocket
(NASA March vacuum)      5% of the Q times the Thrust/PowerInput of a perfectly collimated photon rocket
(Cannae Superconducting) 0.02% to 0.03% of the Q x the Thrust/PowerInput of a perfectly collimated photon rocket
Iulian Berca                   Unknown Q

Notice the very low relative performance of Cannae's superconducting
(very difficult to stay at resonance with a Q > 10^9
« Last Edit: 06/03/2015 08:30 pm by Rodal »

Offline smartcat

  • Member
  • Posts: 10
  • Liked: 7
  • Likes Given: 45

At the moment it's like a bottle full of petrol looking for a live match.. unfortunately it lacks the legs to go looking.

Did you mean unfortunately?  ;D

Offline phaseshift

  • Full Member
  • *
  • Posts: 104
  • Seattle, WA
  • Liked: 84
  • Likes Given: 97

Thus the possibility of quantum tunneling photons being responsible for EM Drive thrust is at play.


I can buy that. The question then becomes how are the frustum's shape and modes responsible for making this happen? How can this be measured?

And if this is the case it sounds horribly dangerous to be behind the thing!



« Last Edit: 06/03/2015 08:30 pm by phaseshift »
"It doesn't have to be a brain storm, a drizzle will often do" - phaseshift

Offline Rodal

  • Senior Member
  • *****
  • Posts: 5911
  • USA
  • Liked: 6124
  • Likes Given: 5564

Thus the possibility of quantum tunneling photons being responsible for EM Drive thrust is at play.


I can buy that. The question then becomes how are the frustum's shape and modes responsible for making this happen? How can this be measured?

And if this is the case it sounds horribly dangerous to be behind the thing!
You need the gradient shape to have photons coming out mainly at one end.  Mode shapes have different amplitude response.

« Last Edit: 06/03/2015 08:34 pm by Rodal »

Offline X_RaY

  • Full Member
  • ****
  • Posts: 852
  • Germany
  • Liked: 1146
  • Likes Given: 2479
@ the case of geometrically attenuation

 Is there a geometrically red- and blue- shift in the cavity causing by change the diameter along the z-axis (topological conditions of the Space Time boundary for the TEM- waves inside a tapered cavity)? The wave gets out of phase while phase velocity is increasing. Red Shifted waves does have a lower energy and therefore a lower momentum (again with respect to the available space). The equivalent space dependent frequency is just different at both endplates were the outer (measurable) frequency is a resultant of an integral over all possible r dependent frequencies by a given fixed length of the cone.
 p=h_quer * k
http://en.wikipedia.org/wiki/Dispersion_relation
That brings us back to different impuse and Poynting vectors..

Dont sure if this is a way caused by well known physics. May be its a way to think about. But that's also confusing for normal brains  :-\
« Last Edit: 06/03/2015 08:50 pm by X_RaY »

Offline deltaMass

  • Full Member
  • ****
  • Posts: 955
  • A Brit in California
  • Liked: 671
  • Likes Given: 275
As has been pointed out, getting Q*photon-rocket thrust might be OK so long as it isn't allowed to do work. Well, much work, anyway. We have equations for that, using the stored energy and the power input.

Offline phaseshift

  • Full Member
  • *
  • Posts: 104
  • Seattle, WA
  • Liked: 84
  • Likes Given: 97

Thus the possibility of quantum tunneling photons being responsible for EM Drive thrust is at play.


I can buy that. The question then becomes how are the frustum's shape and modes responsible for making this happen? How can this be measured?

And if this is the case it sounds horribly dangerous to be behind the thing!
You need the gradient shape to have photons coming out mainly at one end.  Mode shapes have different amplitude response.

Right, but how can 93% of the photons tunnel? What about the shape or modes is causing that to happen?

Wouldn't Shawyer's 1,000,000+ Q superconducting thruster have pretty much melted his table top if photons were just tunneling out?
"It doesn't have to be a brain storm, a drizzle will often do" - phaseshift

Offline sneekmatrix

Wouldn't the tunnelling effect also be constrained by conservation of momentum and therefore apply at both ends of the frustrum?

Offline Rodal

  • Senior Member
  • *****
  • Posts: 5911
  • USA
  • Liked: 6124
  • Likes Given: 5564

Thus the possibility of quantum tunneling photons being responsible for EM Drive thrust is at play.


I can buy that. The question then becomes how are the frustum's shape and modes responsible for making this happen? How can this be measured?

And if this is the case it sounds horribly dangerous to be behind the thing!
You need the gradient shape to have photons coming out mainly at one end.  Mode shapes have different amplitude response.

Right, but how can 93% of the photons tunnel? What about the shape or modes is causing that to happen?

Wouldn't Shawyer's 1,000,000+ Q superconducting thruster have pretty much melted his table top if photons were just tunneling out?
Well, between NASA's results and a private company's reported results I prefer NASA's results, particularly NASA's vacuum result (5% of the Q times ...) but that is just me.  You can choose the private company if you want  :)

(NASA March vacuum)      5% of the Q times the Thrust/PowerInput of a perfectly collimated photon rocket
« Last Edit: 06/03/2015 09:03 pm by Rodal »

Offline Rodal

  • Senior Member
  • *****
  • Posts: 5911
  • USA
  • Liked: 6124
  • Likes Given: 5564
Wouldn't the tunnelling effect also be constrained by conservation of momentum and therefore apply at both ends of the frustrum?
You have to look at the energy density regarding radiation pressure, and don't ignore the lateral conical walls.
Then perform a quantum tunneling analysis.  Momentum will be favored to one side if there is a gradient of emission.
« Last Edit: 06/03/2015 08:55 pm by Rodal »

Offline Rodal

  • Senior Member
  • *****
  • Posts: 5911
  • USA
  • Liked: 6124
  • Likes Given: 5564
As has been pointed out, getting Q*photon-rocket thrust might be OK so long as it isn't allowed to do work. Well, much work, anyway. We have equations for that, using the stored energy and the power input.
Well, I never promised you a rose garden with constant acceleration for constant power.  This thing has been tested at NASA for periods of what ? about 40 seconds ? and people are already extracting conclusions based on constant acceleration for constant power going to the stars, and forgetting about the 2nd Law ???  ;)
« Last Edit: 06/03/2015 09:00 pm by Rodal »

Offline sneekmatrix

As many have said before the EW drive just can't work without violating CoM.  ...
Who has said that "the EM Drive drive just can't work without violating CoM"? I don't know of any author that has advocated that the EM Drive works by violating Conservation of Momentum.  Every single theory I know of, Shawyer, McCulloch, Yang's, White, you name it, postulates that the EM Drive does not violate conservation of momentum (they all have different explanations as to why momentum is conserved).
Perhaps opening the cavity is akin to Schrodinger's cat and the observed thrust is negated. Best to leave the box closed until theory can come up with an answer.

I think you can find the forum is full of extremely knowledgeable people who have said as such.  Yes, the authors you listed have made claims but in each case have opened their system such that CoM appears to not be violated and that's how they make the 'claim' they are not violating basic physics.

Focusing on the question I asked might be more productive.  How did Yang open her system such that CoM does not appear to be violated?  Or do you really believe that the EM Drive, a closed cavity, is not violating CoM without a mechanism for opening it up and therefore in your opinion this line of questioning can lead no where?
Perhaps we are using different languages to express this.

I use the following definition of an open system:

"A open system is a system that has external interactions. Such interactions can take the form of information, energy, or material transfers into or out of the system boundary, depending on the discipline which defines the concept. An open system is contrasted with the concept of an isolated system which exchanges neither energy, matter, nor information with its environment. An open system is also known as a constant volume system or a flow system."

According to that definition Dr. White's theory is opening the system by involving the Quantum Vacuum, if we take that the Quantum Vacuum was not part of the system being considered.

 I think that neither Shawyer, McCulloch or Yang are "opening the system".    I don't think that discussing a gradient of group velocity, or Unruh radiation, or considering current density J is opening the system. But I guess that it all depends on what one means by an open system.

As to what Prof. Yang is doing, I don't completely understand it, but my take (with a grain of salt) is that she maybe considering the case of a cavity coupled externally with a waveguide or a coaxial. Thus, the excitations of a mode in a cavity can be modeled by an equivalent electric ( J ) or magnetic (  Jm ) density current representing the sources of the modes. The equivalent magnetic sources are, for example, the magnetic field on a coupling slot between the waveguide and the cavity and the magnetic field generated by a loop coupled with a cavity, while the equivalent electric sources are the currents on a small antenna coupled with the cavity.

We are using the same definition of open system :)

I do think Shawyer  attempted to "open" the cavity by invoking Special Relativity (incorrectly). White uses the QV (almost in an extra dimensional way) to open the system. Yang appears to make no attempt at doing so and thus the reason for my question.

Going extra dimensional, 4+1, also opens up the cavity. However, in looking over Randall/Sundrum I realized that their +1 is not on our D-Brane and is rather the bulk itself.  According to string theory this means RF energy can not enter the +1 dimension,yet  in the same breath Randall wonders if Standard Model particles are in the bulk.  Of course there is the possibility of other dimensions outside of their theory.

I am hoping that given Yang's substantially higher thrust that the manner in which the cavity is opened up could be discerned helping theory move forward.

From zeno effect wiki

It is easy to show using standard theory that if a system starts in an eigenstate of some observable, and measurements are made of that observable N times a second, then, even if the state is not a stationary one, the probability that the system will be in the same state after, say, one second, tends to one as N tends to infinity; that is, that continual observations will prevent motion …

— Alan Turing as quoted by A. Hodges in Alan Turing: Life and Legacy of a Great Thinker p. 54

« Last Edit: 06/03/2015 09:20 pm by sneekmatrix »

Offline X_RaY

  • Full Member
  • ****
  • Posts: 852
  • Germany
  • Liked: 1146
  • Likes Given: 2479
Wouldn't the tunnelling effect also be constrained by conservation of momentum and therefore apply at both ends of the frustrum?
You have to look at the energy density regarding radiation pressure, and don't ignore the lateral conical walls.
Then perform a quantum tunneling analysis.  Momentum will be favored to one side if there is a gradient of emission.

correct and
The tunneling effect acts instantaneous. At the moment a photon is tunneling it impulse acts, that's like its reflected in a wall <z (lower qua the real length of the cavity). There has to be a blue shift of the signal means higher frequency like calculated r and z dependent.

Offline phaseshift

  • Full Member
  • *
  • Posts: 104
  • Seattle, WA
  • Liked: 84
  • Likes Given: 97
Quantum tunneling also would imply a severe red-shifting from the transfer of energy to momentum.  Seems that a quick calculation should get us in the ball park of the frequency to search for.
"It doesn't have to be a brain storm, a drizzle will often do" - phaseshift

Tags:
 

Advertisement NovaTech
Advertisement Northrop Grumman
Advertisement
Advertisement Margaritaville Beach Resort South Padre Island
Advertisement Brady Kenniston
Advertisement NextSpaceflight
Advertisement Nathan Barker Photography
1